a bag contains Stephanie five marbles, some red some blue. The ratio of red marbles to blue ones is 3:2. how many red marbles are there?

Answers

Answer 1

Answer:

There are 3 red marbles.

Step-by-step explanation:

The ratio of red marbles to blue marbles is 3:2, so for every two blue marbles there are three red marbles. This means that there are three red marbles for every four marbles in the bag. Therefore, if there are five marbles in the bag, there must be three red marbles and two blue marbles.


Related Questions

Let $m$ and $n$ be positive integers such that $m$ has exactly 5 positive divisors, $n$ has exactly 6 positive divisors, and $mn$ has exactly 14 positive divisors. How many distinct prime factors does $mn$ have

Answers

If [tex]A[/tex] is the set of positive divisors of [tex]m[/tex] and [tex]B[/tex] the set of positive divisors of [tex]n[/tex], then [tex]A\cup B[/tex] is the set of positive divisors of [tex]mn[/tex].

Use the inclusion/exclusion principle:

[tex]|A \cup B| = |A| + |B| - |A \cap B|[/tex]

where [tex]|\cdot|[/tex] denotes set cardinality (the number of elements the set contains). The set [tex]A\cap B[/tex] is the set of common divisors of [tex]m[/tex] and [tex]n[/tex]. Then

[tex]14 = 5 + 6 - |A\cap B| \implies |A\cap B| = 3[/tex]

so that [tex]m[/tex] and [tex]n[/tex] share 3 divisors [tex]d_1,d_2,d_3[/tex]; let [tex]k=d_1d_2d_3[/tex] be their product. They must be prime

This means we can write

[tex]m = p_1 p_2 k[/tex]

[tex]n = p_3 p_4 p_5 k[/tex]

[tex]\implies mn = p_1 p_2 p_3 p_4 p_5 k^2 = p_1 p_2 p_3 p_4 p_5 {d_1}^2 {d_2}^2 {d_3}^2[/tex]

so that [tex]mn[/tex] has up to 8 distinct prime factors.

I really need help!!!


Solve each equation by using the Quadratic Formula. Round to the nearest tenth
if necessary.
72. x²-25=0
75. 2r²+r- 14 = 0
73. r²+25=0
76. 5v^2-7v = 1

Answers

Answer:

72. 5

73. -5

74.r =2.40753645…,−2.90753645

75. v =1.53066238…,−0.13066238

Step-by-step explanation:

Please help with this and pls give all steps!!!

Answers

Answer:

No solutions.

Step-by-step explanation:

Problem:

Solve y−4x=−3;y=4x+7

Steps:

I will solve your system by substitution.

y=4x+7;−4x+y=−3

Step: Solve y=4x+7 for y:

Step: Substitute 4x+7 for y in −4x+y=−3:

−4x+y=−3

−4x+4x+7=−3

7=−3(Simplify both sides of the equation)

7+−7=−3+−7(Add -7 to both sides)

0=−10

Answer:

No solutions.

Answer:

  No solutions

Step-by-step explanation:

It is easiest to determine whether the equations are consistent by putting both of them in the same form.

Adjusting the form

The equation on the left is in "standard form." The one on the right is in "slope-intercept form." We can rewrite either one of them to put it into the other form.

Rewriting the left equation to slope-intercept form, we have ...

  y = 4x -3 . . . . . . . add 4x to both sides

Comparing this to the right equation ...

  y = 4x +7

we see that the slopes are the same, and the intercepts are different. These equations describe parallel lines. Parallel lines can never meet, so cannot have any point in common. The equations are inconsistent and have no solution.

Rewriting the right equation to standard form, we have ...

  y -4x = 7 . . . . . . . subtract 4x from both sides

Comparing this to the left equation ...

  y -4x = -3

we see that the coefficients are the same, but the constants are different. There can be no values of x and y that will satisfy both equations. Any values that make the terms have one sum cannot also make them have a different sum.

There are no solutions.

On a quiz show, Linda won $10 less than three times as much as Charlie. If Linda won $800, how much did Charlie win?

Answers

Using a system of equations, it is found that Charlie won $270.

What is a system of equations?

A system of equations is when two or more variables are related, and equations are built to find the values of each variable.

In this problem, the variables are:

Variable x: Amount earned by Linda.Variable y: Amount earned by Charlie.

Linda won $10 less than three times as much as Charlie, hence:

x = 3y - 10

Linda won $800, hence the amount won by Charlie is found as follows:

800 = 3y - 10

3y = 810

y = 810/3

y = 270

More can be learned about a system of equations at https://brainly.com/question/24342899

#SPJ1

Suppose a batch of metal shafts produced in a manufacturing company have a variance of 9 and a mean diameter of 207 inches. If 72 shafts are sampled at random from the batch, what is the probability that the mean diameter of the sample shafts would differ from the population mean by greater than 0.3 inches

Answers

The probability that the mean diameter of the sample shafts would differ from the population mean by greater than 0.3 inches is 39.54%.

Given mean diameter of 207, variance=9, sample size of 72.

We have to calculate the probability that the mean diameter of the sample shafts would differ from the population mean by greater than 0.3 inches.

The sample mean may be greater than or less than from population mean than 0.3 inches.

Either greater than 207+0.3=207.3 inches,

Smaller =207-0.3=206.7

Since the normal distribution is symmetric these probabilities are equal. So we find one of them and multiply by 2.

Probability of being less than 206.7

P value of z when X=206.7. So

Z=(X-μ)/s

=(206.7-207)/0.35

=-0.3/0.35

=-0.857

p value =0.1977

Probability of differing from population mean greater than 0.3 inches=2*0.1977

=0.3954

=39.54%

Hence the probability that the mean diameter of the sample shafts would differ from the population mean  by greater than 0.3 inches is 39.54%.

Learn more about probability at https://brainly.com/question/24756209

#SPJ4

For the arithmetic sequence beginning with the terms {7, 11, 15, 19, 23 ...}, what is the sum of the first 31 terms?

a.)
1827

b.)
1950

c.)
2077

d.)
2208

Answers

Step-by-step explanation:

here a= 7 ,d=4

then S31= 31/2(7+7+30*4)= 31/2(14+120)= 31/2(134)= 31*67= 2077

hope it helps

The sum of the first 31 terms is 2077.

What is Arithmetic Sequence?

An arithmetic sequence is one in which each phrase grows by adding or subtracting some constant k. In contrast, in a geometric sequence, each term grows by dividing/multiplying some constant k.

We have,

Sequence:  {7, 11, 15, 19, 23 ...}

First term = 7

Common difference, d= 11- 7 = 4

Now, the sum of first 31 st term of sequence

= 31/2 [ 2(7) + (31-1)4]

= 31/2 [ 14 + 120]

= 31/2 x 134

= 31 x 67

= 2077

Learn more about Arithmetic Sequence here:

https://brainly.com/question/15412619

#SPJ2

Heheh be svdbtjdjsbddb

Answers

Did u type that by mistake
Well he did become 3x y 6’

The question is on the picture below

Answers

Answer: 54

Step-by-step explanation:

By the inscribed angle theorem, angle ZXY measures half of 108 degrees, which is 54 degrees.

In the triangle below, what is the measure of

Answers

Answer:

D. 30°

Step-by-step explanation:

QP and QR have the same length "8" , therefore, the triangle is an isosceles triangle. Opposite angles are congruent.

- ∠R = ∠P

- ∠R = 30°

-----------------------------

What is the value of X

Answers

Answer:

x = 10

Step-by-step explanation:

BF   is two times AX   so

2 ( 2x+10) = 5x+10

4x + 20 = 5x + 10

x = 10

Find the altitude of this equilateral triangle - geometry

Answers

Answer:

4√3.

Step-by-step explanation:

If the height is h then

tan 60 = h / (1/2*8)

tan 60 = h/4

h = 4 tan 60

   = 4 * √3.

PLEASE HELP ME WITH THIS QUESTION TT ITS A EQUIVALENT QUESTION!!!

thank you!​

Answers

Answer:  [tex]\boldsymbol{\frac{5\text{x}-2}{\text{x}^2-\text{x}}}[/tex]  (choice D)

You have the correct answer.

=====================================================

Work Shown:

[tex]\frac{2}{\text{x}} + \frac{3}{\text{x}-1}\\\\\frac{2(\text{x}-1)}{\text{x}(\text{x}-1)} + \frac{3\text{x}}{\text{x}(\text{x}-1)}\\\\\frac{2\text{x}-2}{\text{x}^2-\text{x}} + \frac{3\text{x}}{\text{x}^2-\text{x}}\\\\\frac{2\text{x}-2+3\text{x}}{\text{x}^2-\text{x}}\\\\ \boldsymbol{\frac{5\text{x}-2}{\text{x}^2-\text{x}}}\\\\[/tex]

--------

Explanation:

The idea I used is that we cannot add the fractions unless the denominators are the same. The denominators x and (x-1) lead to the lowest common denominator, aka LCD, of [tex]\text{x}(\text{x}-1) = \text{x}^2 - \text{x}[/tex]. We multiply the denominators together to get the LCD.

The first original fraction [tex]\frac{2}{\text{x}}[/tex] is missing (x-1) in the denominator. This is why I multiplied top and bottom by (x-1) in the 2nd step. Similarly, the fraction [tex]\frac{3}{\text{x}-1}[/tex] is missing an x out front to get to x(x-1). This is why I multiplied top and bottom of that fraction by x.

After we get the denominators to be the same, we can then add the numerators like any other algebraic expression. The denominator stays the same the entire time.

It's similar to how [tex]\frac{2}{7} + \frac{3}{7} = \frac{2+3}{7} = \frac{5}{7}[/tex] has the numerators add like you'd expect while the denominator stays at 7 the entire time.

You can think of it like this: "2 sevenths + 3 sevenths = 5 sevenths" or "2s+3s = 5s" for short. The term "sevenths" is effectively a unit such as cm or meters. We must have common units if we want to add them.

Greetings. As a beginner, I'm struggling a bit to learn calculus. May I know what is the derivative of x to the power 4 step by step?
Thanks in advance!

Answers

If you're just starting calculus, perhaps you're asking about using the definition of the derivative to differentiate [tex]x^4[/tex].

We have

[tex]\dfrac{d}{dx} x^4 = \displaystyle \lim_{h\to0} \frac{(x+h)^4 - x^4}h[/tex]

Expand the numerator using the binomial theorem, then simplify and compute the limit.

[tex]\dfrac{d}{dx} x^4 = \displaystyle \lim_{h\to0} \frac{(x^4+4hx^3 + 6h^2x^2 + 4h^3x + h^4) - x^4}h \\\\ ~~~~~~~~ = \lim_{h\to0} \frac{4hx^3 + 6h^2x^2 + 4h^3x + h^4}h \\\\ ~~~~~~~~ = \lim_{h\to0} (4x^3 + 6hx^2 + 4h^2x + h^3) = \boxed{4x^3}[/tex]

In general, the derivative of a power function [tex]f(x) = x^n[/tex] is [tex]\frac{df}{dx} = nx^{n-1}[/tex]. (This is the aptly-named "power rule" for differentiation.)

what is the value of x when (fog)(x)=-8

Answers

The value of x when (f o g)(x) = -8 is 3

How to solve for x?

The composite function is given as:

(f o g)(x) = -8

The above is calculated as:

(f o g)(x) = f(g(x))

So, we have:

f(g(x)) = -8

From the table, we have:

f(-4) = -8

This means that:

g(x) = -4

From the ordered pair, we have:

g(x) = -4 when x = 3

Hence, the value of x is 3

Read more about composite functions at:

https://brainly.com/question/10687170

#SPJ1

The value of x when (f o g)(x) = -8 is 3

How to solve for x?

The composite function is given as:

(f o g)(x) = -8

The above is calculated as:

(f o g)(x) = f(g(x))

So, we have:

f(g(x)) = -8

From the table, we have:

f(-4) = -8

This means that:

g(x) = -4

From the ordered pair, we have:

g(x) = -4 when x = 3

Hence, the value of x is 3

Read more about composite functions at:

brainly.com/question/10687170

#SPJ1

A full can of milk weighs 70 pounds. If exactly half of the milk is poured out, it weighs 38 pounds. How much does the empty can weigh?

Answers

Answer:

6 pounds

Step-by-step explanation:

Let the can weigh x pounds

Let the milk weigh y when full, therefore half is [tex]\frac{y}{2}[/tex]

Now we for 2 simultaneous equations:

[tex]x + y =70[/tex] ....(i)

[tex]x+ \frac{y}{2}= 38[/tex] .... (ii)

we solve by eliminating the x in both equation throug subtraction

[tex]x - x + y- \frac{y}{2} =70-38\\y- \frac{y}{2} = 32\\\frac{y}{2} = 32\\y=2(32)\\y=64\\x+y=70\\x+64=70\\x=6[/tex]

2) Express 34m 5cm 6mm in millimeteres​

Answers

Answer:

34079 millimeters

Step-by-step explanation:

__________________________________

I assume you mean millimeters, but

34 m = 34000 millimeter4 cm = 4 centimeters = 40 millimeter5 mm = 5 millimeter

All together adds up to 79 millimeter

34000 + 40 + 5 = 34079 mm

__________________________________

Hope this helped

(Edit) Sorry I didn't see meters

I’m having issues with finding the restriction of the equation in the photo

Answers

Answer:

x = 9

Step-by-step explanation:

Restricted value of the expressionSimplify the equation.Set the denominator to 0.Solve  and find the solution.The solution is the restricted value of the equation.

[tex]\sf \dfrac{(x +7)(x+8)}{(x -9(x + 6)} \ \div \ \dfrac{(x+8)(x - 2)}{(x+ 6)(x - 2)} =\dfrac{(x +7)(x+8)}{(x -9)*(x +6)}*\dfrac{(x + 6)(x - 2)}{(x +8 )(x - 2)}[/tex]

                                                   [tex]\sf =\dfrac{x +7}{x - 9}[/tex]

x - 9 = 0

   x = 9

If we plugin x = 9, then the denominator would become 0 and hence the expression will become undefined.

So, x = 9 is the restricted value of the expression.

which vectors are unit vectors?

Answers

The unit vectors are u = {1, 1}. Option C

What are unit vectors?

Unit vectors can be defined as vector with magnitude of 1.

It is important to note that a unit vector is a vector quantity. Vector quantities are known to have magnitude and direction.

They are represented with the sign 'u'

Thus, the unit vectors are u = {1, 1}. Option C

Learn more about unit vectors here:

https://brainly.com/question/13696986

#SPJ1

What is the equation of the function that is graphed as line a?

Answers

Answer:

Option 1

Step-by-step explanation:

The equations in the options are in the slope-intercept form (y= mx +c, where m is the slope and c is the y-intercept).

In the graph, the line cuts through the y-axis at y= -2. Thus, c= -2 and the 2nd option can be eliminated.

The slope can be found using the formula below.

[tex]\boxed{\text{slope}=\frac{y_1-y_2}{x_1-x_2} }[/tex]

Let's choose 2 points on the line: (0, -2) and (-2, 4)

Slope

[tex]=\frac{4-(-2)}{-2-0}[/tex]

[tex]=\frac{4+2}{-2}[/tex]

[tex]=\frac{6}{-2}[/tex]

= -3

The equation of the line is thus y= -3x -2.

For more questions on equation of line, do check out the following!

https://brainly.com/question/14200719

Solve: (6x2 + 5x + 1) + (x + 2).

Answers

Answer:

6x+6x2+3

Step-by-step explanation:

6x2+5x+1+x+2

Combine 5x and x to get 6x.

6x2+6x+1+2

Add 1 and 2 to get 3.

6x2+6x+3

The simplified form of the expression (6x² + 5x + 1) + (x + 2) is 6x^2 + 6x + 3.

To solve the expression (6x² + 5x + 1) + (x + 2)

we need to combine like terms.

First, let's remove the parentheses:

6x² + 5x + 1 + x + 2

Next, we combine like terms:

6x² + (5x + x) + (1 + 2)

Simplifying further:

6x^2 + 6x + 3

Therefore, the simplified form of (6x² + 5x + 1) + (x + 2) is 6x² + 6x + 3.

To learn more on Expressions click:

https://brainly.com/question/14083225

#SPJ6

Which statement correctly explains how to prove △ABC∼△DEF?

Answers

Answer:

A

Step-by-step explanation:

calculate the ratios AB/DE BC/EF AND AC/DF。their ratios are 1/2

Answer:

Step-by-step explanation:

the first because the ratio is always from a small to a large triangle, and it is right, the second is wrong, the sides are not congruent, the third is also wrong, the last ratio is wrong

The scale on a map is 1 cm : 6 km. If two cities are 13 cm apart on the map, what is the actual distance between the cities?

Answers

Answer:

78 km!

Step-by-step explanation:

13 cm * 6 km per cm = 78 km in actual distance

ASAP please

The sequence of transformations that can be performed on quadrilateral ABCD to show that it is congruent to quadrilateral GHIJ is followed by a--------------.

Answers

The sequence of transformations that can be performed on quadrilateral ABCD to show that it is congruent to quadrilateral GHIJ is a translation followed by a rotation.

What is a transformation?

Transformation is the movement of a point from its initial location to a new location. Types of transformation are reflection, translation, rotation and dilation.

Translation is the movement of a point either up, down, left or right on the coordinate plane.

The sequence of transformations that can be performed on quadrilateral ABCD to show that it is congruent to quadrilateral GHIJ is a translation followed by a rotation.

Find out more on transformation at: https://brainly.com/question/4289712

#SPJ1

Convert 2077 to base eight

Answers

To convert 2077₁₀ to base eight is 4035₈

How to convert 2077 to base 8?

Since 2077 is in base 10, to convert it to base 8, we successively divide it by 8 and keep the remainder. We divide until we have zero as the dividend then count the remainder from bottom to top.

So, to convert 2077 to base 8, we have

2077 ÷ 8 = 259 r 5

259 ÷ 8 = 32 r 3

32 ÷ 8 = 4 r 0

4 ÷ 8 = 0  r 4

So, counting the remainders from bottom to top, we have 4035₈

So, 2077₁₀ = 4035₈

So, to convert 2077₁₀ to base eight is 4035₈

Learn more about convert to base eight here:

https://brainly.com/question/13556214

#SPJ1

-4,12-5-22,24-100,37 ordenar de menor a mayor

Answers

Answer: 24 - 100, 12 - 5 - 22, -4, 37

Step-by-step explanation:

Simplificar:

-4 = -4

12 - 5 - 22 = -15

24 - 100 = -76

37 = 37

Ordenar de menor a mayor:

-76, -15, -4, 37

       \/

24 - 100, 12 - 5 - 22, -4, 37

24 - 100=-76
12 - 5 - 22=-15
- 4, lo mismo
37, lo mismo

La orden es:
-76, -15, -4, 37

Use any method to solve the equation. If necessary, round to the nearest hundredth.
7x^2-16x=8

Answers

Hello,

Answer:

S = { -0,42 ; 2,71 }

Step-by-step explanation:

7x² - 16x = 8 ⇔ 7x² - 16x - 8 = 0

a = 7 ; b = -16 ; c = -8

Δ = b² - 4ac = (-16)² - 4 × 7 × (-8) = 480 > 0

x₁ = (-b - √Δ)/2a = (16 - √480)/14 ≈ -0,42

x₂ =  (-b + √Δ)/2a = (16 + √480)/14 ≈ 2,71

Answer:

The two answers:

2.707778736

0.42206445

Step-by-step explanation:

Here is the equation:

[tex] {7x}^{2} - 16x = 8[/tex]

Take away the 8 from the right hand side, so that we are left with this quadratic equation:

[tex] {7x}^{2} - 16x - 8 = 0[/tex]

This equation is too complex to solve with the factorizing method, so let's use the quadratic formula, which is as follows:

[tex]x = \frac{-b \pm \sqrt{b^2 - 4ac}}{2a}[/tex]

In this equation, a = 7, b = -16, and c = -8. So let's substitute in:

[tex]x = \frac{-( - 16) \pm \sqrt{ { - 16}^{2} - 4 \times 7 \times - 8}}{2 \times 7}[/tex]

[tex]x = \frac{ - ( - 16) \pm \sqrt{256 + 224} }{14}[/tex]

[tex]x = \frac{ 16 \pm \sqrt{480}}{14}[/tex]

And let's work out the two possible answers:.

2.707778736

0.42206445

3
Which expression is equivalent to x-2
O
O O
O
2x-8
13–5x
-5r-8
2x-8
-5x-4
x-2
13–5x
2x-8
-
5
2- 4
x-2
?

Answers

Step-by-step explanation:

[tex] = \frac{ \frac{3}{x - 2} - 5 }{2 - \frac{4}{x - 2} } [/tex]

[tex] = \frac{ \frac{3 - 5.(x - 2)}{x - 2} }{ \frac{2.(x - 2) - 4}{x - 2} } [/tex]

[tex] = \frac{ \frac{3 - 5.(x - 2)}{ \cancel{x - 2}} }{ \frac{2.(x - 2) - 4}{ \cancel{x - 2} }} [/tex]

[tex] = \frac{3 - 5.(x - 2)}{2.(x - 2) - 4} [/tex]

[tex] = \frac{3 - 5x + 10}{2x - 4 - 4} [/tex]

[tex] = \frac{ - 5x + 13}{2x - 8} [/tex]

[tex] = \frac{13 - 5x}{2x - 8} [/tex]

The answer is D.

The equivalent value of the expression is A = ( 13 - 5x ) / ( 2x - 8 )

What is an Equation?

Equations are mathematical statements with two algebraic expressions flanking the equals (=) sign on either side.

In an equation, the expressions on either side of the equals sign are called the left-hand side (LHS) and the right-hand side (RHS), respectively. The equals sign (=) indicates that the two expressions have the same value, and that the equation is true for certain values of the variables involved.

Coefficients, variables, operators, constants, terms, expressions, and the equal to sign are some of the components of an equation. It typically consists mathematical operations, such as addition, subtraction, multiplication, division, and exponentiation.

Given data ,

Let the equation be represented as A

Now , the value of A is

[tex]A = \frac{\frac{3}{x-2}-5}{2\:-\:\frac{4}{x-2}}[/tex]

On simplifying , we get

[tex]A = =\frac{\frac{-5x+13}{x-2}}{2-\frac{4}{x-2}}[/tex]

On further simplification , we get

[tex]A = =\frac{\frac{-5x+13}{x-2}}{\frac{2x-8}{x-2}}[/tex]

Therefore , the value of A is A = ( 13 - 5x ) / ( 2x - 8 )

Hence , the equation is A = ( 13 - 5x ) / ( 2x - 8 )

To learn more about equations click :

https://brainly.com/question/19297665

#SPJ5

PLEASE HELP< GEOMTREY SUCKS!!!!!!!!!

Answers

Answer:

The measure of angle DCB = 125

Step-by-step explanation:

This is because angle ACD and angle DCB are supplementary.

Supplementary angles are two angles with a sum of 180 degrees, and we know these angles are supplementary by the fact that angle ACB = 180 degrees.

There are 480 students in a school.
and
1
of the students wear glasses.
5
How many of the students do not wear glasses

Answers

Answer:

96 students

Step-by-step explanation:

I am not exactly sure if this question is talking about a 1:5 ratio, but that's what I am assuming. To get 96, you need to replace the 5 with 480 because the problem seems to mean 1 to every 5 people wear glasses. Next, you do 480/5 to get the number on the left side of the ratio. You get 96 when you do that. I am a little confused with the question because it isn't exactly formatted correctly, but this is what I am guessing. Hope this helps!

Help I need this urgently!!!!

Answers

it’s -5/3 because the slope is -1.66667 and -5/3 is -1.66667
Other Questions
SCHOOL OF ROCK MOVIEWhat is the movie "School of Rock" criticizing?What is the conflict in the movie?What is the solution for that conflict?What is the problem with school according to the "substitute teacher"? What does he propose for school?How does this experience changes the lives of the teacher and the student? Which of the following is not a reason to choose a community college? 2) a) Calculate the new average temperature of earth using information from Question 1 butin a case where the whole earth is covered with water. what is meant by hydrolysis? Solve the system of equations. Type in all points of intersection for the two functions and round to the nearest tenth if necessary. show workf(x) = - 0.5x + 2g(x) = x^3 - 5x^2 + 3 Adriana and Sophie have summer jobs selling newspaper subscriptions door-to-door, but theircompensation plans are different. Adriana earns a base wage of $6 per hour, as well as $2 forevery subscription that she sells. Sophie gets $3 per subscription sold, in addition to a basewage of $4 per hour. If they each sell a certain number of subscriptions in an hour, they willend up earning the same amount. How much would each one earn?Write a system of equations, graph them, and type the solution. 4. A banking system provides users with several services:(a) to check their accounts to see recent transactions and balances;(b) to pay bills online;(c) to buy and sell investments.All of these services require a secure log-in. If the system crashes or the networkconnection fails during a transaction, all the financial data must be left in a consistentstate.a. Draw use case diagrams for these requirements. Use standard symbols and shapes. MSOffice Visio, Lucidchart, draw.io, Creately, Smartdraw or Figma are optional toolsb. Give the use case specification for paying a bill online. The purpose of statistical inference is to make estimates or draw conclusions about a _____. a. population based upon information obtained from the sample b. mean of the sample based upon the mean of the population c. sample based upon information obtained from the population d. statistic based upon information obtained from the population Vertical lines have an undefined slope.True or False Select the correct answer. This table represents a quadratic function. x y 1 3 3 -3 5 -5 7 -3 9 3 Which statement about this function is true? A. The value of a is negative, so the vertex is a maximum. B. The value of a is positive, so the vertex is a maximum. C. The value of a is positive, so the vertex is a minimum. D. The value of a is negative, so the vertex is a minimum. Reset Next What is the equilibrium constant for the reaction? Which list shows all the factors of 24? A. 1, 2, 3, 4, 6, 8, 12, 24 B. 1, 2, 3, 4, 6, 12, 24 C. 1, 2, 4, 5, 6, 12, 24 D. 1, 2, 4, 6, 12, 24 when he reaches home his sisters his mother and his wife son for a long time at the slight of his flat sleeve in an episode of war by stephan crane what does the lieutenants sleeve represent When two musical notes are a "fifth" apart, the frequency of the lower note is 2/3 the frequency of the higher note. Using f as the frequency of the higher note, write an expression for the frequencyof the lower notes What is the magnitude of the force that must be applied for the system below to be balanced?Group of answer choices20 N0 N80 N30 N Is the relationship between the appearance of the sample and the ntu value of each sample consistent for all samples A firm attempting to reduce the size of its workforce may use several approaches. The approach most likely to be saved as a last resort is What is the range of these weights?79 g88 g116 g109 g93 g Write an equation for the nth term of the given arithmetic sequence. 7, 13, 19, 25 What is a technique to gain personal information for the purpose of identity theft, usually by means of fraudulent emails that look as though they came from legitimate businesses? Pharming Phishing Phishing Expedition. Spear Phishing. g